Relativity and Absolute Space: A Reconciliation?

  • Thread starter pilopais
  • Start date
  • Tags
    Relativity
In summary, Einstein came up with the theory of special relativity due to the non-covariant nature of Maxwell's equations under Galilean transformation. This was in contrast to Newton's laws, which follow the covariant aspect of the equations. With the discovery of the relativistic form of Maxwell's equations and the constant speed of light in all inertial frames, special relativity became an agreement with real experiments.
  • #36
Thrice said:
Like you've been told before, there are an infinite number of theories that are empirically equivalent to SR.
There is absolute simultaneity, there is relative simultaneity, and there are linear combinations of the two.
Can you guess why they fail?
No, I don't care to guess. Please cite a credible reference showing why they fail.
 
Physics news on Phys.org
  • #37
Epsilon Pi said:
Hello ZZ,

Before you close this thread, please it is a real fallacy to bring again that argument with relative velocities, that of course, they can be zero in its own frame, but you cannot do this with the magnetic field, or more properly speaking with the intrinsic magnetic moment of the electron. Please, please, you cannot make zero a magnetic field by a relative movement, this is a faulty consequence of the application of the principle of relativity to Maxwell's equations. On the other hand your reference to materials, that I suppose, are diamagnetic materials with a zero magnetic field, does not have anything to do with relative movements. You have not addressed my point, so we are really talking about different things, sorry.

My best regards

EP

Again with this "intrinsic" magnetic moment of an electron.

Answer me this once and for all : How does this intrinsic magnetic moment of an electron have anything to do with Maxwell equation? For example, such magnetic moment cannot be DERIVED out of any Maxwell equation (i.e. it is NOT due to any motion of charge!). Thus, you have no frame of reference to transformation to.

Secondly, take ANY dipole field. BY ITS GEOMETRY, you do not have a frame to transform to in which all directions of the magnetic field disappears.

Neither of those have any conflict with SR!

Thirdly, and this is where you have it wrong, take a linear line current producing the standard circuital magnetic field. This magnetic field is due to the MOTION of the charge, NOT due to the "intrinsic magnetic moment of each electron". Are you claiming that by transforming to the inertial frame that these electrons are stationary, that the result where this magnetic field disappears is NOT valid?

I will not go into the ridiculous insistance that one can actually MEASURE the net magnetic field of a glob of electron due to their individual spins in a conductor or even in an electron beam. However, for some odd reason, you seem to think that the more DOMINANT magnetic field due to moving charges somehow cannot disappear simply because the individual net spin don't. This is what I found astounding, considering that you still haven't told me what is the net magnetic field eminating from your computer keyboard due to all those "spins".

Zz.
 
  • #38
These are quantum examples. It isn't really fair to expect anything less than a quantum theory to explain intrinsic moment of an elementary particle, although if you consider a classically-scaled version (eg. a solenoid) then there shouldn't be any problems.
 
  • #39
Epsilon Pi said:
Hello ZZ,

Before you close this thread, please it is a real fallacy to bring again that argument with relative velocities, that of course, they can be zero in its own frame, but you cannot do this with the magnetic field, or more properly speaking with the intrinsic magnetic moment of the electron. Please, please, you cannot make zero a magnetic field by a relative movement, this is a faulty consequence of the application of the principle of relativity to Maxwell's equations. On the other hand your reference to materials, that I suppose, are diamagnetic materials with a zero magnetic field, does not have anything to do with relative movements. You have not addressed my point, so we are really talking about different things, sorry.

My best regards

EP

Why not consider something other than electrons? How about a [tex]\pi^+[/tex] particle? It has positive charge, but is spin-0, so it has no magnetic dipole.

If a pion is at rest, there will not be a magnetic field. If on the other hand, the pion is moving, there will be. As has been pointed out repeatedly, we can easily transform between these frames; and, we must get exactly the results I specified. This, of course, means that the identity of electric and magnetic fields is frame dependent. However, the acceleration of a second particle due to the force exerted on it by the pion will not be frame dependent. It's simply the identification of the cause of that acceleration that changes.
 
  • #40
Relativity & Absolutism?

Oh dear, dear, dear, I "see" you so entangled with the linear-philosophical-mathematical abstraction of SR that it is almost impossible to have the possibility to consider another point of view?

With the magnetic field we have a relation to velocity that is not linear at all, as is expressed mathematically in the vector product found in the Lorentz force equation, that as a matter of fact, determines sort of "absolute" direction of space, why insisting then in its relativity with a linear-uniform translatory motion?

And finally you have brought another good example of what I want to express with "intrinsic" when you wrote and asked:

ZapperZ said:
Answer me this once and for all : How does this intrinsic magnetic moment of an electron have anything to do with Maxwell equation? For example, such magnetic moment cannot be DERIVED out of any Maxwell equation (i.e. it is NOT due to any motion of charge!). Thus, you have no frame of reference to transformation to.

Yes, it's an intrinsic magnetic property of the electron, it cannot be deduced or it should not be deduced from more basic theoretical principles, why should we deduce such a FACT? why such a futile intent? why not using a methodology where both induction and deduction are considered at the same time?... but there we have that Maxwell's equation about the non existence, in the physical space out there, of magnetic monopoles, why insisting in dipoles with the magnetic field then?... But then you will have the tendency to deduce the existence of something that is intrinsic from charge? No, my friend, it cannot be deduced from electric charge, on the contrary there where we have a moving charge, a current, there we have a magnetic field around, this is what is expressed precisely by Ampere's law, one of the Maxwell's equations. The question is: can we have "an electron" at rest? Is not this a question that should not be asked according to the Uncertainty principle? On the other hand, as Einstein pointed out with understanding the electron it is enough.

My best regards

EP



ZapperZ said:
Again with this "intrinsic" magnetic moment of an electron.

Answer me this once and for all : How does this intrinsic magnetic moment of an electron have anything to do with Maxwell equation? For example, such magnetic moment cannot be DERIVED out of any Maxwell equation (i.e. it is NOT due to any motion of charge!). Thus, you have no frame of reference to transformation to.

Secondly, take ANY dipole field. BY ITS GEOMETRY, you do not have a frame to transform to in which all directions of the magnetic field disappears.

Neither of those have any conflict with SR!

Thirdly, and this is where you have it wrong, take a linear line current producing the standard circuital magnetic field. This magnetic field is due to the MOTION of the charge, NOT due to the "intrinsic magnetic moment of each electron". Are you claiming that by transforming to the inertial frame that these electrons are stationary, that the result where this magnetic field disappears is NOT valid?

I will not go into the ridiculous insistance that one can actually MEASURE the net magnetic field of a glob of electron due to their individual spins in a conductor or even in an electron beam. However, for some odd reason, you seem to think that the more DOMINANT magnetic field due to moving charges somehow cannot disappear simply because the individual net spin don't. This is what I found astounding, considering that you still haven't told me what is the net magnetic field eminating from your computer keyboard due to all those "spins".

Zz.
 
  • #41
Epsilon Pi said:
Oh dear, dear, dear, I "see" you so entangled with the linear-philosophical-mathematical abstraction of SR that it is almost impossible to have the possibility to consider another point of view?

With the magnetic field we have a relation to velocity that is not linear at all, as is expressed mathematically in the vector product found in the Lorentz force equation, that as a matter of fact, determines sort of "absolute" direction of space, why insisting then in its relativity with a linear-uniform translatory motion?

And finally you have brought another good example of what I want to express with "intrinsic" when you wrote and asked:
Yes, it's an intrinsic magnetic property of the electron, it cannot be deduced or it should not be deduced from more basic theoretical principles, why should we deduce such a FACT? why such a futile intent? why not using a methodology where both induction and deduction are considered at the same time?... but there we have that Maxwell's equation about the non existence, in the physical space out there, of magnetic monopoles, why insisting in dipoles with the magnetic field then?... But then you will have the tendency to deduce the existence of something that is intrinsic from charge? No, my friend, it cannot be deduced from electric charge, on the contrary there where we have a moving charge, a current, there we have a magnetic field around, this is what is expressed precisely by Ampere's law, one of the Maxwell's equations. The question is: can we have "an electron" at rest? Is not this a question that should not be asked according to the Uncertainty principle? On the other hand, as Einstein pointed out with understanding the electron it is enough.

My best regards

EP

But this is silly! There's nothing contradictory about SR regarding an electron's spin and magnetic moment. The SR's equation DO NOT PREDICT that you can transform to a frame in which the electron spin vanishes, contrary to your claim! So your original objection is faulty and moot in the first place!

What it does claim is that in a case where you have charges moving with a constant velocity, you can transform to an inertial frame whereby the magnetic field due to the moving charges is no longer present!

When you apply things in the wrong fashion, you can't blame that thing for being wrong. You can solve this very easily by showing the exact derivation here that produces the faulty result. If you can do that correctly, I'll tell you where to send it in for publication so that you can show the rest of the world that physicists have gotten it WRONG all these while with E&M. After call, the classical E&M that you claim to be so familiar with will ALSO have problems with your example of the application of the HUP. How come you didn't come here to refute THAT?

Of all the strange and weird objections that I have heard all this while regarding SR (and trust me, I've heard some really strange ones all these years), this HAS to be the silliest of them all. You are claiming something that SR never claimed. And if you look closely, what you think is the shortcoming of SR is really a shortcoming of Maxwell Equations! They themselves are the ones to be "blamed" for having nothing on an electron's intrinsic magnetism - this is purely quantum mechanical.

And thank you Parlyne for bringing up the pion. Excellent example. I should have thought of that earlier.

You still have not answered MY question. What is the magnetic moment of a clump of glass or the plastic that make up your keyboard.

Zz.
 
  • #42
Yes, I think I'm losing my time, and I'm sorry but I've now more important things to do than to involve myself is sort of metaphysical, philosophical matter. How can you divorce, as a physicist, electron's spin and magnetic moment from its real source an inherent magnetic field? Yes, I know you'll then introduce ad hoc fields that does not have anything to do with physical reality.

No, no, you cannot cancel a magnetic fiels by a relative movement, this is really the wrong prediction of SR!

You wrote and asked:

ZapperZ said:
You still have not answered MY question. What is the magnetic moment of a clump of glass or the plastic that make up your keyboard.

Sorry you are the expert in materials -specially in diamagnetic materials- you should know the answer, but it does not have anything to do with the point in question, the cancelation of a magnetic field by a relative movement, I'm sure of it.

My best regards and thank you so much for your time and effort, but I won't go on with this discussion going to nowhere. Please I'm short of time, specially when the only thing you have to say is that I'm writing silly things, the typical reaction when there is no real argument to bring.

My best wishes with your philosophical-metaphysical speculations about the application of the principle of relativity!

My best regards

EP

ZapperZ said:
But this is silly! There's nothing contradictory about SR regarding an electron's spin and magnetic moment. The SR's equation DO NOT PREDICT that you can transform to a frame in which the electron spin vanishes, contrary to your claim! So your original objection is faulty and moot in the first place! I'l switch to no email notification, so there will be no answer anymore to this thread.



What it does claim is that in a case where you have charges moving with a constant velocity, you can transform to an inertial frame whereby the magnetic field due to the moving charges is no longer present!

When you apply things in the wrong fashion, you can't blame that thing for being wrong. You can solve this very easily by showing the exact derivation here that produces the faulty result. If you can do that correctly, I'll tell you where to send it in for publication so that you can show the rest of the world that physicists have gotten it WRONG all these while with E&M. After call, the classical E&M that you claim to be so familiar with will ALSO have problems with your example of the application of the HUP. How come you didn't come here to refute THAT?

Of all the strange and weird objections that I have heard all this while regarding SR (and trust me, I've heard some really strange ones all these years), this HAS to be the silliest of them all. You are claiming something that SR never claimed. And if you look closely, what you think is the shortcoming of SR is really a shortcoming of Maxwell Equations! They themselves are the ones to be "blamed" for having nothing on an electron's intrinsic magnetism - this is purely quantum mechanical.

And thank you Parlyne for bringing up the pion. Excellent example. I should have thought of that earlier.

You still have not answered MY question. What is the magnetic moment of a clump of glass or the plastic that make up your keyboard.

Zz.
 
  • #43
Epsilon Pi said:
My best wishes with your philosophical-metaphysical speculations about the application of the principle of relativity!

My best regards

EP

Next time you and the people you love fly in an airplane, just remember that all of you put your LIVES on the validity Special Relativity. And THAT is a FACT.

Zz.
 
  • #44
Epsilon Pi said:
... but there we have that Maxwell's equation about the non existence, in the physical space out there, of magnetic monopoles, why insisting in dipoles with the magnetic field then?
You can write-in magnetic charge to Maxwell's equations, and they would still be consistent with all known experiments. This is typically left out only because no experiment has detected it yet.
 
Last edited:
  • #45
Aether said:
Lorentz ether theory (LET, aka GGT) is empirically equivalent to SR, and there are no failed experiments associated with it.
Mickelson-Morley searching for an Ether and changing speed of light but showing no evidence of it or a preferred reference frame.
Plus others that are the equivalent qualify as failing to confirm

There needs to be something to support a “preferred frame” or even a ‘relative’ “preferred frame” to use LET/GGT.
 
  • #46
RandallB said:
Mickelson-Morley searching for an Ether and changing speed of light but showing no evidence of it or a preferred reference frame.
Plus others that are the equivalent qualify as failing to confirm
These tests ruled out other ether theories, but they did not rule out LET/GGT.
There needs to be something to support a “preferred frame” or even a ‘relative’ “preferred frame” to use LET/GGT.
We haven't detected a real locally preferred frame yet, but we haven't ruled out the possibility that one exists either. You can use any arbitrarily preferred frame with LET/GGT.
 
  • #47
RandallB said:
There needs to be something to support a “preferred frame” or even a ‘relative’ “preferred frame” to use LET/GGT.
True, but LET followers believe that it is indetectable because of the Lorentz factor applied to distance and duration on the object in motion relative to the preferred frame.
 
  • #48
Hey !, hello to all,

Being a layperson, I’m very excited about the arguments / counterarguments that are everywhere in these forums. They are all an immense source of new information about the physics and the people trying to understand and explain how such physics work.

So, Zz, what you are saying is that, if there is a permanent magnet lying on the same table as a lab experiment setup with moving charges in a conductor, there will be a difference of perceived resulting magnetic fields if I were to move at the speed of the moving charges and look at the table. One field will remain and the other one will vanish… is this right ?


VE
 
  • #49
Hello VE,

Being you a layperson, you have put it quite simple and quite right, but I don't think you'll receive any answer, as an answer, regarding the non-vanishing of a magnetic field by a relative movement, would mean to admit that the principle of relativity is not valid when applied to Maxwell's equations, the greatest drawback of SR.

My best regards

EP
ValenceE said:
Hey !, hello to all,

Being a layperson, I’m very excited about the arguments / counterarguments that are everywhere in these forums. They are all an immense source of new information about the physics and the people trying to understand and explain how such physics work.

So, Zz, what you are saying is that, if there is a permanent magnet lying on the same table as a lab experiment setup with moving charges in a conductor, there will be a difference of perceived resulting magnetic fields if I were to move at the speed of the moving charges and look at the table. One field will remain and the other one will vanish… is this right ?


VE
 
  • #50
ValenceE said:
So, Zz, what you are saying is that, if there is a permanent magnet lying on the same table as a lab experiment setup with moving charges in a conductor, there will be a difference of perceived resulting magnetic fields if I were to move at the speed of the moving charges and look at the table. One field will remain and the other one will vanish… is this right ?
To reproduce with an electric current in a conductor the magnetic field generated by a permanent magnet, we should use a solenoid. What happens in the reference frame of the moving charges inside the conductor is not obvious at all, because they are not in an inertial ref. frame; worst, they are rotating in a spiral!
It's not possible to say simply that the magnetic field wouldn't vanish.
 
  • #51
Yes, the point is that, that concept of an inertial ref. frame is such an idealization, that it does not work at all, in that real life of rotating systems... then why we insist in them?

Just wondering, wondering and wondering

My best regards

EP


lightarrow said:
To reproduce with an electric current in a conductor the magnetic field generated by a permanent magnet, we should use a solenoid. What happens in the reference frame of the moving charges inside the conductor is not obvious at all, because they are not in an inertial ref. frame; worst, they are rotating in a spiral!
It's not possible to say simply that the magnetic field wouldn't vanish.
 
  • #52
VE, you are quite correct. In the reference frame of the particles in the current, there will be no magnetic field due to that current - just an electric field. However, there will still be a magnetic field due to the bar magnet. That field, however, will not be exactly the same as the one seen in the lab frame. And, in fact, there will also be an electric field associated with the bar magnet.

EP, what I have stated here is exactly the result predicted by special relativity. In fact, this is what is meant by saying that the E and B fields are only defined relative to a given reference frame. It means that the fields due to a given set of sources depend also on the velocity of the observer relative to the sources.
 
  • #53
ValenceE said:
Hey !, hello to all,

Being a layperson, I’m very excited about the arguments / counterarguments that are everywhere in these forums. They are all an immense source of new information about the physics and the people trying to understand and explain how such physics work.

So, Zz, what you are saying is that, if there is a permanent magnet lying on the same table as a lab experiment setup with moving charges in a conductor, there will be a difference of perceived resulting magnetic fields if I were to move at the speed of the moving charges and look at the table. One field will remain and the other one will vanish… is this right ?VE

The issue here, some that EP doesn't seem to understand, is the geometry of the magnetic field.

Consider the magnetic field generated by a beam of charges moving with a constant velocity v. Now solve the Maxwell Equation to get the magnetic field. This magnetic field has a very particular geometry. Now, transform yourself to the frame of reference moving with the charges. You can now look at it in two different ways:

1. You see the "source" of the fields, being the charges, at rest in your frame, and thus, you ONLY see an electric field. This is no different than you having a line of charge in the first place. Any undergrad physics student can solve for such a field.

2. You don't look at the "source", but rather act on the field. In this case, you do a tranformation of the magnetic field to the moving frame, and you will see that you get the same E-field.

This shows that 1 and 2 are consistent with each other. The principle of symmetry is one of the fundamental principles of our universe.

Now, this is where we get back to your question. If you have the same charges moving, and you have, let's say a bar magnet, what happens if you do the SAME transformation of the field? The bar magnet will NOT lose its magnetic field in the moving frame, not even by applying the relativistic Maxwell equation! Why? Becuase the geometry of the field is all "wrong" - it isn't identical to the field created by the moving charges. You will get a different geometry from the bar magnet in the moving frame depending on how it is alligned, but not the "disappearing" field that you get with the line source.

This is why I said early on that the geometry of the field is extremely important. Something like a dipole field can't be transformed away to an inertial frame. The geometry of the field won't work, and this is what you would get when applying the relativistic maxwell equation.

Zz.
 
  • #54
Hello my dear friend ZZ,

You wrote:

ZapperZ said:
Consider the magnetic field generated by a beam of charges moving with a constant velocity v. Now solve the Maxwell Equation to get the magnetic field. This magnetic field has a very particular geometry.

In this case, I suppose you are referring to Ampere's law, that says that around a current of electrons, there is a magnetic field around. This is what you understand by solving Maxwell equation, right? If I interpret things like this I don't find any problem with the geometry of the magnetic field, and again this solution does not have anything to do with SR.

But now, you speculate,

ZapperZ said:
Now, transform yourself to the frame of reference moving with the charges. You can now look at it in two different ways:

1. You see the "source" of the fields, being the charges, at rest in your frame, and thus, you ONLY see an electric field. This is no different than you having a line of charge in the first place. Any undergrad physics student can solve for such a field.

2. You don't look at the "source", but rather act on the field. In this case, you do a tranformation of the magnetic field to the moving frame, and you will see that you get the same E-field.

This shows that 1 and 2 are consistent with each other. The principle of symmetry is one of the fundamental principles of our universe.

and speculate, because here you are not making any reference to Maxwell equation but just to the principle of relativity, please note that for your argument here you are making an abstraction in the concept of charge, you are not taking or thinking in the real origin of both the charge and the magnetic field, I mean, the electron. I really cannot follow you here, please, you cannot do this with the real currents we deal everyday!

My Best regards

EP



ZapperZ said:
The issue here, some that EP doesn't seem to understand, is the geometry of the magnetic field.

Consider the magnetic field generated by a beam of charges moving with a constant velocity v. Now solve the Maxwell Equation to get the magnetic field. This magnetic field has a very particular geometry. Now, transform yourself to the frame of reference moving with the charges. You can now look at it in two different ways:

1. You see the "source" of the fields, being the charges, at rest in your frame, and thus, you ONLY see an electric field. This is no different than you having a line of charge in the first place. Any undergrad physics student can solve for such a field.

2. You don't look at the "source", but rather act on the field. In this case, you do a tranformation of the magnetic field to the moving frame, and you will see that you get the same E-field.

This shows that 1 and 2 are consistent with each other. The principle of symmetry is one of the fundamental principles of our universe.

Now, this is where we get back to your question. If you have the same charges moving, and you have, let's say a bar magnet, what happens if you do the SAME transformation of the field? The bar magnet will NOT lose its magnetic field in the moving frame, not even by applying the relativistic Maxwell equation! Why? Becuase the geometry of the field is all "wrong" - it isn't identical to the field created by the moving charges. You will get a different geometry from the bar magnet in the moving frame depending on how it is alligned, but not the "disappearing" field that you get with the line source.

This is why I said early on that the geometry of the field is extremely important. Something like a dipole field can't be transformed away to an inertial frame. The geometry of the field won't work, and this is what you would get when applying the relativistic maxwell equation.

Zz.
 
  • #55
Epsilon Pi said:
In this case, I suppose you are referring to Ampere's law, that says that around a current of electrons, there is a magnetic field around. This is what you understand by solving Maxwell equation, right? If I interpret things like this I don't find any problem with the geometry of the magnetic field, and again this solution does not have anything to do with SR.

No, it doesn't have anything to do with relativity. Relativity is only necessary when we want to know what the fields that someone moving through the lab at constant speed would measure the fields to be.

and speculate, because here you are not making any reference to Maxwell equation but just to the principle of relativity, please note that for your argument here you are making an abstraction in the concept of charge, you are not taking or thinking in the real origin of both the charge and the magnetic field, I mean, the electron. I really cannot follow you here, please, you cannot do this with the real currents we deal everyday!

My Best regards

EP

Actually, he is making quite a bit of reference to the Maxwell equations. Perhaps it would help to make it more explicit.

An electrical current consists of a collection of charges with some coherent motion (generally much smaller than the random motions of the individual electrons). We can express this idea with the formula:

[tex]I = \lambda q v_d[/tex]

Or, in english, current is equal to the number of charged particles per unit length multiplied by the charge of one particle and the drift speed. The drift speed is the magnitude of the average of all the particle velocities.

If, instead of just sitting in the lab frame, we were to move parallel to the wire at speed [tex]v_d[/tex], we would observe that there was no net motion of electrons in the wire and, hence, no current. We know from Ampere's law (or, if you feel like making things really ugly, the Bios-Savart law) that there should be a magnetic field due to the motion of the charges, when there is such a motion. This means that in the lab frame, a magnetic field should be observed, while in the moving frame one should not be.

If we use a relativistic description of the fields, we will find that this same result holds just by transforming between the frames, instead of having to consider what happens to the sources. And, were we to do this explicitly, we would find that not only is are the magnetic fields different in the two frames; but, the electric fields are, as well. This can be attributed to the difference in charge density due to the Lorentz-Fitzgerald contraction.

In the cases of a current loop or a bar magnet, there is no frame we could transform to in which there is no net motion of charges (or equivalent motion, in the case of the bar magnet). This means that there is no frame with 0 magnetic field. However, as ZZ pointed out, the fields will not be the same when measured in two different frames; they will just not be 0 in either.
 
  • #56
Thank you, Parlyne for being more explicit!

Parlyne said:
If, instead of just sitting in the lab frame, we were to move parallel to the wire at speed [tex]v_d[/tex], we would observe that there was no net motion of electrons in the wire and, hence, no current. We know from Ampere's law (or, if you feel like making things really ugly, the Bios-Savart law) that there should be a magnetic field due to the motion of the charges, when there is such a motion. This means that in the lab frame, a magnetic field should be observed, while in the moving frame one should not be.

Sorry, my friend, but I really, do not understand this, from the physical point of view, unless you are thinking in a though experiment. I've never seen a current vanished in the real life by such a relative movement, this is just an idealization, a result of applying the principle of relativity to something that is not relative: the inherent magnetic field of the electron.

Oh God, how you can say you can cancel a current by a relative movement? Do you realize the implication this would have if it were true?

My best regards

EP
 
  • #57
For a current loop which is circular, IS possible to find a ref. frame in which the electrons are stationary: a frame in the center, rotating with the same electron's angular speed. Of course, it's not an inertial ref. frame, however.
I was answering to Parlyne.
 
Last edited:
  • #58
Epsilon Pi said:
Thank you, Parlyne for being more explicit!



Sorry, my friend, but I really, do not understand this, from the physical point of view, unless you are thinking in a though experiment. I've never seen a current vanished in the real life by such a relative movement, this is just an idealization, a result of applying the principle of relativity to something that is not relative: the inherent magnetic field of the electron.

Oh God, how you can say you can cancel a current by a relative movement? Do you realize the implication this would have if it were true?

My best regards

EP

Have you ever tried to measure the magnetic field of a wire through which no current is travelling? If not, try it. You'll find that there is no magnetic field. This is not all that different from the case of moving along a wire at the drift velocity of the electrons.

And, no one here except you is trying to apply relativity to the magnetic dipole of an electron. Why? Because the magnetic field from the current in a wire is not caused by the electrons' dipole moments.
 
  • #59
lightarrow said:
For a current loop which is circular, IS possible to find a ref. frame in which the electrons are stationary: a frame in the center, rotating with the same electron's angular speed. Of course, it's not an inertial ref. frame, however.
I was answering to Parlyne.

True, but we can't use Lorentz transformations to get to that frame, so the question of what happens to the fields under Lorentz transformations can't be answered that way.
 
  • #60
Sorry a wire, through which no current is traveling, or a wire where there is no electrons?

An electrons' dipole moments? what is it? I know the electron has an intrinsic magnetic moment, or an inherent polarity that gives reason of that intrinsic magnetic moment, and intrinsic means it cannot be vanished by a relative movement.

My best regards

EP
Parlyne said:
Have you ever tried to measure the magnetic field of a wire through which no current is travelling? If not, try it. You'll find that there is no magnetic field. This is not all that different from the case of moving along a wire at the drift velocity of the electrons.

And, no one here except you is trying to apply relativity to the magnetic dipole of an electron. Why? Because the magnetic field from the current in a wire is not caused by the electrons' dipole moments.
 
  • #61
A slight error in this discussion is that since a wire is electrically neutral, electrons really move relative to the positive ions; so in a frame where the electrons are at rest, there will be positive ions moving in the opposite direction.

For this reason, I recommend we use a beam of charged particles and look at that in different frames, as opposed to a wire.
 
  • #62
Epsilon Pi said:
Sorry a wire, through which no current is traveling, or a wire where there is no electrons?

An electrons' dipole moments? what is it? I know the electron has an intrinsic magnetic moment, or an inherent polarity that gives reason of that intrinsic magnetic moment, and intrinsic means it cannot be vanished by a relative movement.

My best regards

EP

Take a stationary line charge. Now unless you are also claiming that Gauss's law is also wrong and an "idealization", find all the field due to that line charge.

Now, someone else is moving with velocity v in the direction that this line charge is oriented. This person sees instead, a moving line of charge, i.e. a current! Thus, via Maxwell equation alone, that person detects a magnetic field.

Now, which part of those two do you not understand, or don't you agree?

Thirdly, NO ONE is claiming that one can transform away a dipole field. You keep asking for the magnetic moment due to the electron spin to be transformed away. Even transforming the relativistic maxwell equations do not result in that! So stop with this nonsense already.

Lastly, and again, this has been mentioned earlier, is that you keep forgetting that we DO transform such a thing in particle accelerators. There are many instances where the dynamics, especially when the particles are undergoing several beam "acrobatics", is simpler when it is solved in the particle's reference frame! In such a condition, the beam self-energy from the magnetic field is transformed away and allows for many of the dynamics to be solved. We then tranform back to the lab frame and use the result! If such a thing is wrong or an "idealization", we would have a ton of wrong results that simply will not match our experiments.

I have asked you before to cite valid references to back your claim. You produced no such thing. I believe that we have been MORE than patience and given you more than enough opportunity for you to air your "opinion" on this matter dispite of your continuing violation of the speculative personal theory guidelines that you have agreed to.

Therefore, this thread is done, and nothing of this nature can be discussed in the main physics forum. Any further issues related to this can only be done in the IR forum.

Zz.
 

Similar threads

  • Special and General Relativity
2
Replies
50
Views
2K
  • Special and General Relativity
6
Replies
185
Views
8K
  • Special and General Relativity
2
Replies
38
Views
2K
  • Special and General Relativity
Replies
10
Views
2K
  • Special and General Relativity
2
Replies
52
Views
4K
  • Special and General Relativity
Replies
6
Views
1K
  • Special and General Relativity
2
Replies
57
Views
4K
  • Special and General Relativity
3
Replies
78
Views
8K
Replies
1
Views
219
  • Special and General Relativity
Replies
24
Views
2K
Back
Top